Complicated System of Equations

Algebra Level 5

Consider the distinct complex solutions ( x 1 , y 1 ) , . . . , ( x n , y n ) (x_1,y_1),...,(x_n,y_n) to the following system of equations: { x 3 + y 2 + y = 1 x 2 + y 3 = 2. \begin{cases} x^3+y^2+y=1\\ x^2+y^3=2. \end{cases} Find the absolute value of the product of all the possible x x values, i.e. x 1 x 2 . . . x n . \left| x_1\cdot x_2\cdot...\cdot x_n \right| .

Details and assumptions

The absolute value of a complex number z = a + b i z = a+ bi is given by z = a 2 + b 2 |z| = \sqrt{a^2+b^2} .

Due to the restriction of distinct solutions, repeated roots (if they exist) do not contribute to the product. For example, if your roots are ( 2 , 3 ) , ( 2 , 3 ) (2, 3), (2, 3) then the answer is 2. If your roots are ( 2 , 3 ) , ( 2 , 3 ) (2, 3), (2, -3) then the answer is 4.


The answer is 11.

This section requires Javascript.
You are seeing this because something didn't load right. We suggest you, (a) try refreshing the page, (b) enabling javascript if it is disabled on your browser and, finally, (c) loading the non-javascript version of this page . We're sorry about the hassle.

12 solutions

Etienne Vouga
Aug 4, 2013

There's probably a simpler way, but here's a brute-force approach.

Solve the first equation for y, then plug it into the second equation and simply, you will get an expression of the form f ( x ) = ± ( 8 4 x 3 ) 5 4 x 3 f(x) = \pm (8-4x^3) \sqrt{5-4x^3} where f(x) is cubic in x. Square both sides, which preserves the number of solutions due to the plus-or-minus, and then simplify to get a degree 9 polynomial. The absolute value of the product of the roots is then the absolute value of the constant term divided by the leading term.

The only thing left to check is that there are no double roots, which can be done by checking that the GCD of the polynomial and its derivative is 1.

Moderator note:

This solution commits a several common mistakes made when trying to solve simultaneous equations by substitution:

  1. There is no guarantee that the set of solutions that you get must satisfy your initial conditions. See Proof that 0 = 1 0=1 .

  2. Since you only solved the equation for x x , you have not shown that there are valid solutions for y y . An implicit assumption made is that for each value of x x , there exists a unique value of y y which satisfies the conditions. The existence and uniqueness is not immediately obvious.

Both of these issues can be worked out, but they have to be acknowledged (see the comments). There does not seem to be a substantially simpler way of solving this problem: it all boils down to a degree 9 9 polynomial (or degree 8 8 , since x = 1 x=-1 is a root).

Since y 2 + y + ( x 3 1 ) = 0 y^2+y+(x^3-1)=0 , 0 = y 6 + y 3 + ( x 3 1 ) 3 3 y 3 ( x 3 1 ) 0=y^6+y^3+(x^3-1)^3-3y^3(x^3-1)

(this follows from the fact that if a + b + c = 0 a+b+c=0 , a 3 + b 3 + c 3 3 a b c = ( a + b + c ) ( a 2 + b 2 + c 2 a b a c b c ) = 0 a^3+b^3+c^3-3abc=(a+b+c)(a^2+b^2+c^2-ab-ac-bc)=0 ).

From the second equation we know that y 3 = 2 x 2 y^3=2-x^2 , so ( 2 x 2 ) 2 + ( 2 x 2 ) + ( x 3 1 ) 3 3 ( 2 x 2 ) ( x 3 1 ) = 0 (2-x^2)^2+(2-x^2)+(x^3-1)^3-3(2-x^2)(x^3-1)=0 .

Collecting the constant, we know from Vieta's formulas that the product of the roots is 2 2 + 2 1 3 ( 2 ) ( 1 ) = 11 2^2+2-1-3(2)(-1)=\boxed{11} .

Edit: Yes Calvin L., sorry about that typo.

Cody Johnson - 7 years, 10 months ago

Log in to reply

Is there a trick for seeing there are no double roots?

Etienne Vouga - 7 years, 10 months ago

Log in to reply

Y'know, I'm not really sure about that. Fortunately Brilliant doesn't require the rigor that competitions such as USAMO do.

Cody Johnson - 7 years, 10 months ago

Note that you should have ( x 3 1 ) 3 (x^3-1)^3 in your third line, which gives a degree 9 polynomial.

Calvin Lin Staff - 7 years, 10 months ago

@Challenge Master I believe your comment is incorrect. Every step in the above is reversible, in particular it can be shown that every solution of f ( x ) 2 = g ( x ) 2 f(x)^2 = g(x)^2 is also a solution of f ( x ) = ± g ( x ) , f(x) = \pm g(x), and vice-versa.

Etienne Vouga - 7 years, 10 months ago

@Challenge Master I believe your second comment is also incorrect.

Once again, *every * step in the above manipulation is reversible.

Belaboring the point: let x be a root of the degree 9 polynomial.

Then

f ( x ) 2 = ( 8 4 x 3 ) 2 ( 5 4 x 3 ) . f(x)^2 = (8-4x^3)^2(5-4x^3).

Then there exists a k in {-1,1} for which f ( x ) = k ( 8 4 x 3 ) 5 4 x 3 . f(x) = k(8-4x^3)\sqrt{5-4x^3}.

Then x 2 + [ 1 + k 1 4 ( x 3 1 ) 2 ] 3 = 2. x^2 + \left[\frac{-1+k\sqrt{1-4(x^3-1)}}{2}\right]^3=2. Then, setting y equal to the stuff in brackets, which is a well-defined complex number, (x,y) satisfies equation 2.

That (x,y) also satisfies equation 1 follows immediately, either by direct verification, or the quadratic formula.

I have no doubt there is slicker solution (Oscar's below is a good step in that direction). But my approach is rigorous, if laborious.

Etienne Vouga - 7 years, 10 months ago

Log in to reply

In addition to there being no double roots, I suppose one must check that the two starting equations do not have a common divisor of degree > 0. Although if they did this question would not be well-posed ;)

Etienne Vouga - 7 years, 10 months ago

That is precisely my point. You need to have explained why any solution x i x_i to the degree 9 polynomial will yield exactly one ( x i , y i ) (x_i, y_i ) pair. How do you know that you didn't introduce any solutions when you proceed to "square both sides"?

(Judging from what you are saying, I'm guessing that) your substitution of y y is y = 1 2 ± 5 4 x 3 2 y = -\frac{1}{2} \pm \frac{ \sqrt{ 5-4x^3} } { 2} . Doe this mean that for every x x value, we get 2 y y values? For example, with x = 1 x = 1 , do we get y = 0 y = 0 and y = 1 y = 1 as solutions?

Your solution is very light on details. It will be helpful to state what f ( x ) f(x) is, instead of simply claiming that it is a cubic polynomial. This will make it easier to follow.

Calvin Lin Staff - 7 years, 10 months ago

Log in to reply

The post that you've replied to proves, rigorously, that for every root x of the degree 9 polynomial, there exists at least one y such that (x,y) is a solution to the original system.

Informally, the same can be seen by the fact that I never divided by any polynomial, squared anything that didn't have a \pm in front of it, or did any other irreversible operation.

Since the degree-9 polynomial has no double roots, the original system has at least nine solutions (all with distinct x). By Bezout's theorem, it has exactly nine.

Etienne Vouga - 7 years, 10 months ago

Log in to reply

@Etienne Vouga I've edited the comment slightly, because I realized that there was an typo in Oscar's solution.

As you said, there "exists at least one y such that (x, y) is a solution to the original system". How do you know that there are not 2 (or more) possible values of y y ? If such a possibility exists, then you need to actively track what happens for each value.

Calvin Lin Staff - 7 years, 10 months ago

Log in to reply

@Calvin Lin Well, there definitely can't be more than 2 ;) We also know (Bezout's theorem) that a system of two degree three polynomial equations has either infinitely many (if they have a nontrivial GCD, which is not the case here) or at most nine solution. Since nine are already accounted for, you can't have more.

Etienne Vouga - 7 years, 10 months ago

Fair enough. I wanted to go back and add more but it won't let me edit :(

Starting from x 2 + [ 1 ± 1 4 ( x 3 1 ) 2 ] 3 = 2 x^2 + \left[\frac{-1\pm\sqrt{1-4(x^3-1)}}{2}\right]^3=2 clear out the denominator and perform the cubing to get 8 x 2 + 3 ± 5 4 x 3 3 ( 5 4 x 3 ) ± ( 5 4 x 3 ) 5 4 x 3 = 16 , 8x^2 + -3 \pm \sqrt{5-4x^3} -3(5-4x^3)\pm(5-4x^3)\sqrt{5-4x^3} = 16, then collect the radicals on one side to get 12 x 3 + 8 x 2 32 = ± ( 8 4 x 3 ) 5 4 x 3 . 12x^3+8x^2-32=\pm(8-4x^3)\sqrt{5-4x^3}.

Etienne Vouga - 7 years, 10 months ago
Wei Liang Gan
May 20, 2014

From the first equation, y 2 + y + ( x 3 1 ) = 0 y^2+y+(x^3-1)=0 and solving the quadratic equation in y y , we get y = 1 ± 1 2 4 ( x 3 1 ) 2 = 1 ± 5 4 x 3 2 y=\frac{-1 \pm \sqrt{1^2-4(x^3-1)}}{2} = \frac{-1 \pm \sqrt{5-4x^3}}{2} .

We then substitute this value of y y into the second equation. We get x 2 + ( 1 ± 5 4 x 3 2 ) 3 = 2 8 x 2 + ( 1 ± 5 4 x 3 ) 3 16 = 0 x^2+(\frac{-1 \pm \sqrt{5-4x^3}}{2})^3 = 2 \Rightarrow 8x^2 + (-1 \pm \sqrt{5-4x^3})^3 - 16 = 0 8 x 2 + ( ( 1 ) ± 3 5 4 x 3 3 ( 5 4 x 3 ) ± ( 5 4 x 3 ) 5 4 x 3 ) 16 = 0 \Rightarrow 8x^2 + ((-1) \pm 3\sqrt{5-4x^3} - 3(5-4x^3) \pm (5-4x^3)\sqrt{5-4x^3}) - 16 = 0 and bringing the square root terms to one side we get 12 x 3 + 8 x 2 32 = ± ( 8 4 x 2 ) 5 4 x 3 12x^3+8x^2-32 = \pm(8-4x^2)\sqrt{5-4x^3} 3 x 3 + 2 x 2 8 = ± ( 2 x 3 ) 5 4 x 3 \Rightarrow 3x^3+2x^2-8=\pm(2-x^3)\sqrt{5-4x^3} ( 3 x 3 + 2 x 2 8 ) 2 = ( ± ( 2 x 3 ) 5 4 x 3 ) 2 \Rightarrow (3x^3+2x^2-8)^2=(\pm(2-x^3)\sqrt{5-4x^3})^2 ( 3 x 3 + 2 x 2 8 ) 2 = ( 2 x 3 ) 2 ( 5 4 x 3 ) \Rightarrow (3x^3+2x^2-8)^2=(2-x^3)^2(5-4x^3)

Note that if we expand and shift the terms to one side, we get a degree 9 polynomial with a leading coefficient of 4 and a constant term of 44. As the complex solutions are required, no extra or wrong solutions are generated from the above algebraic manipulations and thus the product of the possible x x values is the product of the roots of the polynomial so using Vieta's Formula, the required answer is 44 4 = 11 |-\frac{44}{4}|=11

Anh Huy Nguyen
May 20, 2014

$$ x^3+y^2+y=1\ \ (1) \wedge x^2+y^3 =2\ \ (2) \Rightarrow y^3=2-x^2 $$ Consider (1): $$ (1)\Leftrightarrow x^3+y^2+y+1=2\Leftrightarrow x^3+\dfrac{y^3-1}{y-1}=2 $$ $$ \Leftrightarrow x^3+\dfrac{1-x^2}{y-1}=2 \Leftrightarrow y-1=\dfrac{1-x^2}{2-x^3} \Leftrightarrow y=\dfrac{3-x^2-x^3}{2-x^3} $$ Hence $$2-x^2=y^3=\left( \dfrac{3-x^2-x^3}{2-x^3} \right)^3 $$ $$\Leftrightarrow (x^3+x^2-3)^3+(2-x^2)(2-x^3)^3=0\ \ (*)$$ Using Vieta's theorem, suppose $$x 1,x 2,...,x n$$ are the roots of the system of equation given, we have $$|x 1x 2...x n|=-((-3)^3+2^4)=11.$$

黎 李
May 20, 2014

x^8-x^7+x^6-4 x^5+7 x^4-6 x^3+3 x^2-11*x+11=0

Abhishek De
May 20, 2014

We know that for any complex number z , z . z = z 2 z, z.\overline{z}=|z|^2 . Again, by the complex conjugate root theorem if z z is a root of the system then z \overline{z} is also a root. Also, a b c . . . = a . b . c . . . |abc...|=|a|.|b|.|c|... . So, basically the problem boils down to: "Eliminate y y , express the system solely as polynomial, f f on x x and the product of the roots is our answer." Using familiar factorization, y 3 1 = ( y 1 ) ( y 2 + y + 1 ) y^3-1=(y-1)(y^2+y+1) , we get, f ( x ) = x 11 x 9 3 x 8 + 3 x 7 + 4 x 6 6 x 5 9 x 4 + 3 x 3 + 19 x 2 11 = 0 f(x)=x^{11}-x^9-3x^8+3x^7+4x^6-6x^5-9x^4+3x^3+19x^2-11=0 But there's a catch; multiple roots ! If \exists multiple roots then f ( α ) = f ( α ) = 0 f(\alpha)=f'(\alpha)=0 . Using this, we can get we have no multiple roots. So answer is 11 \boxed{11} since the last coefficient gives the product of the roots.

Ziwei Lu
May 20, 2014

Slightly manipulate the first equation, we have y 2 + y + 1 = 2 x 3 y^2+y+1=2-x^3 . Now we can multiply both sides by ( y 1 ) (y-1) to get y 3 1 = ( 2 x 3 ) ( y 1 ) y^3-1=(2-x^3)(y-1) , which is much easier to work with than the original first equation. The equation can then be further manipulated into y 3 1 = ( 2 x 3 ) y 2 + x 3 y^3-1=(2-x^3)y-2+x^3

From the second equation, we have y 3 = 2 x 2 y^3=2-x^2 , or y = 2 x 2 3 y=\sqrt[3]{2-x^2} . Substitute this back into the equation from above, we have 1 x 2 = ( 2 x 3 ) 2 x 2 3 2 + x 3 1-x^2=(2-x^3)\sqrt[3]{2-x^2}-2+x^3 , or 3 x 2 x 3 = ( 2 x 3 ) 2 x 2 3 3-x^2-x^3=(2-x^3)\sqrt[3]{2-x^2} . Cube both sides of the equation, and moving everything to the left to have ( 3 x 2 x 3 ) 3 ( 2 x 3 ) 3 ( 2 x 2 ) = 0 (3-x^2-x^3)^3-(2-x^3)^3(2-x^2)=0

We can see the left side is a 11th degree polynomial. As we are looking for the product of roots, we are interested in the leading coefficient, which is 1 -1 , and the constant term, which is 27 16 = 11 27-16=11 . Thus the product of the roots is 11.

First we show that y 0 y\neq{0} . If y = 0 y=0 clearly the equations are not satisfied.

Now the idea is to extract from the above equations a polynomial of x, the roots of which comprise the set of x solutions to the system. To that purpose, we multiply the first equation by y.

x 3 + y 2 + y = 1 x 3 y + y 3 + y 2 = y x^{3}+y^{2}+y=1\Rightarrow{x^{3}y+y^{3}+y^{2}=y}

y 3 = 2 x 2 y^{3}=2-x^{2}

y 2 = 1 y x 3 y^{2}=1-y-x^{3} .

Substituting the last 2 expressions into the first, we obtain a relation that is linear in terms of y, and therefore easily solvable.

y = x 3 + x 2 3 x 3 2 y=\frac{x^{3}+x^{2}-3}{x^3-2} .

Upon back substitution, the desired polynomial for x, in implicit form is:

x 2 + ( x 3 + x 2 3 x 3 2 ) 3 = 2 x^{2}+\left(\frac{x^{3}+x^{2}-3}{x^3-2}\right)^{3}=2

After verifying that x 3 2 x^{3}\neq{2} , we have that:

x 2 ( x 3 2 ) 3 + ( x 3 + x 2 3 ) 3 = 2 ( x 3 2 ) 3 x^{2}(x^{3}-2)^{3}+(x^{3}+x^{2}-3)^{3}=2(x^{3}-2)^{3}

We know that the absolute product of all roots of every polynomial is the absolute value of its constant term. In this case, we can easily calculate that the constant term is -11. So the answer is 11.

Note: if x 3 = 2 x^{3}=2 then

y 2 + y + 1 = 0 y = 1 ± i 3 2 y^{2}+y+1=0\Rightarrow{y=\frac{-1\pm{i\sqrt{3}}}{2}} , which does not satisfy both equations.

Jimmi Simpson
May 20, 2014

From the second equation, it can be shown that y = ( 2 x 2 ) 1 / 3 y=\left(2-x^2\right)^{1/3} . Plugging this into the first equation yields ( 2 x 2 ) 2 / 3 + ( 2 x 2 ) 1 / 3 = \left(2-x^2\right)^{2/3}+\left(2-x^2\right)^{1/3} = 1 x 3 1-x^3 (let this be equation 1). Factoring on the left-hand side yields ( 2 x 2 ) 1 / 3 ( ( 2 x 2 ) 1 / 3 + 1 ) = \left(2-x^2\right)^{1/3}\left(\left(2-x^2\right)^{1/3}+1\right)= 1 x 3 1-x^3 . Both sides are then cubed: ( 2 x 2 ) ( x 2 + 3 ( 2 x 2 ) 2 / 3 + 3 ( 2 x 2 ) 1 / 3 + 3 ) = \left(2-x^2\right)\left(-x^2+3\left(2-x^2\right)^{2/3}+3\left(2-x^2\right)^{1/3}+3\right) = ( 1 x 3 ) 3 \left(1-x^3\right)^3 . But, from equation 1, substitution yields x 2 + 3 ( 2 x 2 ) 2 / 3 + 3 ( 2 x 2 ) 1 / 3 + 3 = -x^2+3\left(2-x^2\right)^{2/3}+3\left(2-x^2\right)^{1/3}+3 = x 2 + 3 ( 1 x 3 ) + 3 = -x^2+3\left(1-x^3\right)+3 = 6 x 2 3 x 3 6 - x^2 - 3x^3 . So all that is left is finding the product of the roots of ( 2 x 2 ) ( 6 x 2 3 x 3 ) = \left(2-x^2\right)\left(6-x^2-3x^3\right) = ( 1 x 3 ) 3 \left(1-x^3\right)^3 . Expansion yields the polynomial 0 = x 9 3 x 6 + 3 x 5 + x 4 3 x 3 8 x 2 + 11 0=x^9-3x^6+3x^5+x^4-3x^3-8x^2+11 . From Vieta's formula, the absolute value of the product of the roots is 11 / 1 = 11 11/1 = 11 .

Sriram Muthuraman
May 20, 2014

x^3+y^2+y=1, find y in terms of x, y=(-1+sqrt(5+4x^3))/2 or y=(-1-sqrt(5+4x^3)/2 but 2nd one is the soln because at x=1 ,y=0,so real soln will be there. now substiute the y in x^2+y^3=2, now construct a polynomial in x, and hence product of roots =11

Gaurav Bansal
May 20, 2014

x^3+y^3=2 so y=(2-x^3)^(1/3) by putting this value in equation 1 we got x^3+(2-x^3)^(2/3)+(2-x^3)^(1/3)=1 (2-x^3)^(2/3)+(2-x^3)^(1/3)=1-x^3 cube both side & after simplifyng expression we get x^9+x^6-11x^3+11=0 so product of all x is 11

x^3+y^2+y=1 is a quadratic in y solve for y we get y=-(1+(sqrt(1-(4x^3)))) ignoring other root because for that if we substitute back we get x=1 as solution which is not possible as roots are complex hence now substitute this y in second equation and square it and simplify we get 9th degree equation in x product of roots is the constant term which is 11 hence answer

David Altizio
May 20, 2014

Notice that the system of equations can be rearranged to get { y 2 + y + 1 = 2 x 3 y 3 1 = 1 x 2 \begin{cases}y^2+y+1 &= 2-x^3\\y^3 - 1 &= 1-x^2\end{cases} Since from the second equation y 3 1 = ( y 1 ) ( y 2 + y + 1 ) = ( y 1 ) ( 2 x 3 ) = 1 x 2 , y^3-1=(y-1)(y^2+y+1)=(y-1)(2-x^3)=1-x^2, we have that y 1 = 1 x 2 2 x 3 y-1=\dfrac{1-x^2}{2-x^3} and y = 3 x 2 x 3 2 x 3 y=\dfrac{3-x^2-x^3}{2-x^3} . Note that it is possible to divide by 2 x 3 2-x^3 since if 2 x 3 = 0 2-x^3=0 , then 1 x 2 1-x^2 (the right-hand side of the equation) must be equal to 0 0 as well, and there is no number x x such that both x 3 = 2 x^3=2 and x 2 = 1 x^2=1 . Substituting this into the original second equation gives [\begin{align*}x^2+\left(\dfrac{3-x^2-x^3}{2-x^3}\right)^3&=2\x^2(2-x^3)^3+(3-x^2-x^3)^3&=2(2-x^3)^3\ x^2-2)(2-x^3)^3+(3-x^2-x^3)^3&=0\end{aligned} By Vieta's Formulas, the absolute value of the product of the roots of any polynomial equation is equal to the absolute value of the result when the constant term of the polynomial is divided by its leading coefficient. First, it may be deduced that the leading coefficient of the LHS is 1 -1 . This is because the left product of polynomials has the only possible x 11 x^{11} term on the LHS, and multiplying it out shows that the coefficient on this term is 1 -1 . Next, it can be deduced that since the constant term of ( x 2 2 ) ( 2 x 3 ) 3 (x^2-2)(2-x^3)^3 is 2 2 3 = 16 -2\cdot 2^3=-16 and the constant term of ( 3 x 2 x 3 ) 3 (3-x^2-x^3)^3 is 3 3 = 27 3^3=27 , the constant term of the entire polynomial is 16 + 27 = 11 -16+27=11 . Therefore, by representing the roots of the original system as x 1 , x 2 , , x 11 x_1,x_2,\ldots,x_{11} we have that x 1 x 2 x 3 x 11 = 11 1 = 11 |x_1x_2x_3\cdots x_{11}=\left|\dfrac{11}{-1}\right|=11 and we are done.

0 pending reports

×

Problem Loading...

Note Loading...

Set Loading...